Đến nội dung

Hình ảnh

[TOPIC] ÔN THI BẤT ĐẲNG THỨC $\boxed{\text{THPT CHUYÊN}}$ LỚP $10$ năm $2018-2019$

bất đẳng thức holder cosi bunhiacopxki

  • Chủ đề bị khóa Chủ đề bị khóa
Chủ đề này có 318 trả lời

#81
HelpMeImDying

HelpMeImDying

    Trung sĩ

  • Thành viên
  • 108 Bài viết

Bài 30: Cho $x,y,z$ là các số thực dương thỏa mãn $xy+yz+xz=xyz$. Chứng minh rằng $\frac{x}{y^{2}}+\frac{y}{z^{2}}+\frac{z}{x^{2}}\geq 3\left ( \frac{1}{x^{2}}+\frac{1}{y^{2}}+\frac{1}{z^{2}} \righ

Đặt $\frac{1}{x}=a; \frac{1}{y}=b; \frac{1}{z}=c\Rightarrow a+b+c= 1$

Bất đẳng thức $\Leftrightarrow \frac{b^{2}}{a}+\frac{c^{2}}{b}+\frac{a^{2}}{c}\geq 3(a^{2}+b^{2}+c^{2})$

Áp dụng AM-GM: $\frac{b^{2}}{a}+9ab^{2}\geq 6b^{2}$

Thiết lập các bđt tương tự thì bđt cần chứng minh trở thành

$3(a^{2}+b^{2}+c^{2})\geq 9(ab^{2}+bc^{2}+ca^{2})\Leftrightarrow (a^{2}+b^{2}+c^{2})(a+b+c)\geq 3(ab^{2}+bc^{2}+ca^{2})\Leftrightarrow a^{3}+b^{3}+c^{3}+a^{2}b+b^{2}c+c^{2}a\geq 2(ab^{2}+bc^{2}+ca^{2})$

Đúng do: $a^{3}+ac^{2}\geq 2a^{2}c$

                $b^{3}+ba^{2}\geq 2ab^{2}$

                $c^{3}+cb^{2}\geq 2c^{2}b$



#82
tr2512

tr2512

    Thượng sĩ

  • Thành viên
  • 272 Bài viết

Bài 23

Cho các số thực không âm a, b, c mà trong đó không có 2 số nào đồng thời bằng 0.

Chứng minh: 

$\frac{a}{{b + c}} + \frac{b}{{a + c}} + \frac{c}{{a + b}} + \frac{{4{\rm{a}}bc}}{{\left( {a + b} \right)\left( {b + c} \right)\left( {c + a} \right)}} \ge 2$ (Schur)

Cái này siêu basic nên mình làm cho vui thôi :D

$VT-VP$=$\frac{{{x^3} + {y^3} + {z^3} + 3{\rm{x}}yz - xy\left( {x + y} \right) - yz\left( {y + z} \right) - z{\rm{x}}\left( {z + x} \right)}}{{\left( {x + y} \right)\left( {y + z} \right)\left( {z + x} \right)}} \ge 0$ (Schur) :D 



#83
PhanThai0301

PhanThai0301

    Trung sĩ

  • Thành viên
  • 167 Bài viết

Bài 35: Cho $\left\{\begin{matrix} x,y,z>0 & & \\ \frac{1}{x}+\frac{1}{y}+\frac{1}{z}=4 \end{matrix}\right.$ Tìm GTLN của biểu thức:

                       P= $\frac{1}{\alpha x+\beta y+\gamma z}+\frac{1}{\beta x+\gamma y+\alpha z}+\frac{1}{\gamma x+\alpha y+\beta z}$ với $\alpha ,\beta ,\gamma \in N^{*}$.


"IF YOU HAVE A DREAM TO CHASE,NOTHING NOTHING CAN STOP YOU"_M10

                                                                                                            


#84
Khoa Linh

Khoa Linh

    Thiếu úy

  • Thành viên
  • 601 Bài viết

Bài 35: Cho $\left\{\begin{matrix} x,y,z>0 & & \\ \frac{1}{x}+\frac{1}{y}+\frac{1}{z}=4 \end{matrix}\right.$ Tìm GTLN của biểu thức:

                       P= $\frac{1}{\alpha x+\beta y+\gamma z}+\frac{1}{\beta x+\gamma y+\alpha z}+\frac{1}{\gamma x+\alpha y+\beta z}$ với $\alpha ,\beta ,\gamma \in N^{*}$.

alpha, beta, gamma lằng nhằng quá :)) mình thay bằng a,b,c. Ta có:

$\frac{1}{ ax+b y+c z}=\frac{\frac{1}{a}+\frac{1}{b}+\frac{1}{c}}{(ax+by+cz)\left ( \frac{1}{a}+\frac{1}{b}+\frac{1}{c} \right )}\leq \left ( \frac{1}{a}+\frac{1}{b}+\frac{1}{c} \right )\left ( \frac{1}{\sqrt{x}+\sqrt{y}+\sqrt{z}} \right )^2$

Tương tự thì 

$P\leq 3\left ( \frac{1}{a}+\frac{1}{b}+\frac{1}{c} \right )\left ( \frac{1}{\sqrt{x}+\sqrt{y}+\sqrt{z}} \right )^2\leq \frac{3}{81}\left( \frac{1}{a}+\frac{1}{b}+\frac{1}{c} \right )\left ( \frac{1}{\sqrt{x}}+\frac{1}{\sqrt{y}}+\frac{1}{\sqrt{z}} \right )^2$

Mặt khác:

$\sum \frac{1}{\sqrt{x}}=\sum \frac{\sqrt{\frac{1}{x}.\frac{4}{3}}}{\sqrt{\frac{4}{3}}}\leq \frac{\sum\frac{1}{x}+4 }{2\sqrt{\frac{4}{3}}}=2\sqrt{3}$

Suy ra P$\leq\frac{4}{9}\left (\frac{1}{a}+\frac{1}{b}+\frac{1}{c} \right )$

p/s: Bạn xem lại điểm rơi nhé sợ mình làm sai nhưng ý tưởng là vậy 


Bài viết đã được chỉnh sửa nội dung bởi Khoa Linh: 18-04-2018 - 20:38

$\sqrt[LOVE]{MATH}$

"If I feel unhappy, I do mathematics to become happy. If I am happy, I

 

do mathematics to keep happy" - Alfréd nyi 


#85
buingoctu

buingoctu

    Thượng sĩ

  • Thành viên
  • 213 Bài viết

Bài 36: Cho a,b,c dương thỏa mãn a+b+c=4.CMR:

$\sqrt[4]{a^3}+\sqrt[4]{b^3}+\sqrt[4]{c^3}>2\sqrt{2}$

Nguồn: Đề thi tuyển sinh tỉnh Vĩnh Phúc 2012-2013


Bài viết đã được chỉnh sửa nội dung bởi buingoctu: 18-04-2018 - 21:07


#86
DOTOANNANG

DOTOANNANG

    Đại úy

  • ĐHV Toán Cao cấp
  • 1609 Bài viết

37. Với các số $a_{1}\,,\, a_{2}\,,\,...\,,\,a_{n}$ là hoán vị của tập $\left \{ 1\,; \,2\,;\, 3\,;\, ...\,;\, 2018 \right \}$. Tìm GTLN của $a_{1}\,a_{2}+ a_{2}\,a_{3}+ ...+ a_{2017}\,a_{2018}$



#87
badaosuotdoi

badaosuotdoi

    Binh nhất

  • Thành viên mới
  • 47 Bài viết

Bài 29: Cho $a,b,c>0$ và $a+b+c=3$. Chứng minh rằng $\frac{a^{2}+bc}{a+bc}+\frac{b^{2}+ac}{b+ac}+\frac{c^{2}+ab}{c+ab}\geq 3$

Ta có BĐT $\Leftrightarrow \sum \frac{a^{2}-a}{a+bc}\geq 0$.....$\sum \frac{3a^{2}-3a}{a+bc}=\sum \frac{a((a-b)+(a-c))}{a+bc}=\sum \frac{(a-b)^{2}c(a+b)}{(a+bc)(b+ac)}\geq 0$..



#88
badaosuotdoi

badaosuotdoi

    Binh nhất

  • Thành viên mới
  • 47 Bài viết

Bài 30: Cho $x,y,z$ là các số thực dương thỏa mãn $xy+yz+xz=xyz$. Chứng minh rằng $\frac{x}{y^{2}}+\frac{y}{z^{2}}+\frac{z}{x^{2}}\geq 3\left ( \frac{1}{x^{2}}+\frac{1}{y^{2}}+\frac{1}{z^{2}} \right )$.

Ta có BĐT $\Leftrightarrow \sum_{cyc}x^{3}z^{2}\geq 3\sum x^{2}y^{2}\Leftrightarrow \sum_{cyc}x^{3}z^{2}(\frac{1}{x}+\frac{1}{y}+\frac{1}{z})\geq 3\sum x^{2}y^{2}$

$\Leftrightarrow \sum_{cyc}(y^{3}x+\frac{z^{3}y^{2}}{x})\geq 2\sum x^{2}y^{2}$...Dễ thấy $\sum_{cyc}\frac{z^{3}y^{2}}{x}\geq 2\sum x^{2}y^{2}-xyz(x+y+z),\sum_{cyc}y^{3}x\geq xyz(x+y+z)$.....$\rightarrow$ đpcm



#89
hoangkimca2k2

hoangkimca2k2

    Sĩ quan

  • Thành viên
  • 474 Bài viết

Bài 24: Cho $a,b,c$ là các số thực thỏa mãn $a^{2}+b^{2}+c^{2}\neq 0$. Chứng minh rằng $\sum_{cyc}\frac{a^{2}-bc}{2a^{2}+b^{2}+c^{2}}\geq 0$

 

Đăng bài giải :D

Cách 1: Dùng tương đương thần chưởng =))

Ta có: $\sum_{cyc}\frac{a^{2}-bc}{2a^{2}+b^{2}+c^{2}}=\sum_{cyc}\frac{(a-c)(a+b)+(a-b)(a+c)}{2a^{2}+b^{2}+c^{2}}=\sum_{cyc}(a-c)\left ( \frac{a+b}{2a^{2}+b^{2}+c^{2}}-\frac{b+c}{2a^{2}+b^{2}+c^{2}} \right )$

$=\frac{(a-c)^{2}(a^{2}+b^{2}+c^{2}-ab-bc-ac)}{(2a^{2}+b^{2}+c^{2})(2c^{2}+b^{2}+a^{2})}\geq 0$ Bất đẳng thức này luôn đúng.

 

Cách 2: 

Bất đẳng thức cần chứng minh tương đương $-\frac{2a^{2}-2bc}{2a^{2}+b^{2}+c^{2}}+1-\frac{2b^{2}-2ac}{2b^{2}+a^{2}+c^{2}}+1-\frac{2c^{2}-2ab}{2c^{2}+a^{2}+b^{2}}+1\geq 3$$\Leftrightarrow \sum_{cyc}\frac{(a+b)^{2}}{2c^{2}+b^{2}+a^{2}}\leq 3$

Áp dụng $Cauchy-schwarz$ ta có: $\frac{b^{2}}{a^{2}+b^{2}}+\frac{c^{2}}{a^{2}+c^{2}}\geq \frac{(b+c)^{2}}{2a^{2}+b^{2}+c^{2}}$. Tương tự cộng vế theo vế có $dpcm$.


  N.D.P 

#90
MarkGot7

MarkGot7

    Hạ sĩ

  • Thành viên
  • 67 Bài viết

Bài 36: Cho a,b,c dương thỏa mãn a+b+c=4.CMR:

$\sqrt[4]{a^3}+\sqrt[4]{b^3}+\sqrt[4]{c^3}>2\sqrt{2}$

Nguồn: Đề thi tuyển sinh tỉnh Vĩnh Phúc 2012-2013

https://i.pinimg.com...7e4a089987e.jpg

 Nhân cả hai vế với $\sqrt{2}$ , ta được:

  $\sqrt[4]{4a^{3}}+ \sqrt[4]{4b^{3}}+ \sqrt[4]{4c^{3}}> 4$ (*)

$\Rightarrow$ Cần chứng minh (*):

 Vì $a+b+c=4 \Rightarrow VT\doteq \sqrt[4]{(a+b+c)a^{3}} + \sqrt[4]{(a+b+c)b^{3}} + \sqrt[4]{(a+b+c)c^{3}}$

Vì $a, b,c > 0$ $\Rightarrow$ VT $> \sqrt[4]{a^{4}}+ \sqrt[4]{b^{4}}+ \sqrt[4]{c^{4}} \doteq a+b+c\doteq 4= VP \Rightarrow (*)\Rightarrow$

đpcm


Bài viết đã được chỉnh sửa nội dung bởi MarkGot7: 18-04-2018 - 22:26

Cuộc đời lắm chông gai thử thách. Chỉ khi ta cố gắng vượt qua, ta mới biết chân quý những thứ mình có được. :icon12:  :icon12:  :icon12:  %%- 


#91
doraemon123

doraemon123

    Trung sĩ

  • Thành viên
  • 169 Bài viết

Bài 38:
IMG_20180418_224653.jpg
P/s: Xin lỗi m.n mình đang dùng đt nên không gõ Latex được


Bài viết đã được chỉnh sửa nội dung bởi doraemon123: 19-04-2018 - 12:56

$\sqrt{MF}$  math is like reality that so many problem to solve $\sqrt{MATH}$

                                               (~~) (~~) :ukliam2: :ukliam2: :ukliam2: :ukliam2:  (~~) (~~) 


#92
thanhdatqv2003

thanhdatqv2003

    Trung sĩ

  • Thành viên
  • 159 Bài viết

Ta có bổ đề quen thuộc $\sum \frac{a}{b+c-a} \geq 3$

$\sum \frac{1-2a^2}{(1-2a)^2}=\sum \frac{1-4a^2}{(1-2a)^2}+\frac{2a^2}{(1-2a)^2}$
$\sum \frac{1-4a^2}{(1-2a)^2}=\sum \frac{(a+b+c)^2-4a^2}{(b+c-a)^2}=\sum \frac{b+c+3a}{b+c-a}=3+\sum \frac{4a}{b+c-a }\geq 15$

$\sum \frac{2a^2}{(b+c-a)^2} \geq \sum \frac{2}{3}(\sum \frac{a}{b+c-a})^2\geq 6$
$-> P \geq 21$


:ohmy: [Không tồn tại các nghiệm nguyên khác không x, y, và z thoả mãn xn + yn = zn trong đó n là một số nguyên lớn hơn 2.  (FERMAT)  :ohmy: 

 

 

 

 


#93
thanhdatqv2003

thanhdatqv2003

    Trung sĩ

  • Thành viên
  • 159 Bài viết

Chứng minh bổ đề như thế nào vậy b

 

Bạn nào ns rõ đc cách chứng minh thì càng tốt


Bài viết đã được chỉnh sửa nội dung bởi MoMo123: 19-04-2018 - 11:49

:ohmy: [Không tồn tại các nghiệm nguyên khác không x, y, và z thoả mãn xn + yn = zn trong đó n là một số nguyên lớn hơn 2.  (FERMAT)  :ohmy: 

 

 

 

 


#94
PhanThai0301

PhanThai0301

    Trung sĩ

  • Thành viên
  • 167 Bài viết

Bài 39: Cho $\left\{\begin{matrix} x, y ,z >0 & & \\ xyz=1 & & \end{matrix}\right.$, chứng minh rằng:

                     P= $\frac{\sqrt{m+x^{3}+y^{3}}}{xy}+\frac{\sqrt{m+x^{3}+y^{3}}}{yz}+\frac{\sqrt{m+z^{3}+z^{3}}}{zx}\geq 3\sqrt{3}$ với $m\epsilon N^{*}$.


"IF YOU HAVE A DREAM TO CHASE,NOTHING NOTHING CAN STOP YOU"_M10

                                                                                                            


#95
hoangkimca2k2

hoangkimca2k2

    Sĩ quan

  • Thành viên
  • 474 Bài viết

Chứng minh bổ đề như thế nào vậy b

 

Bạn nào ns rõ đc cách chứng minh thì càng tốt

em không nên spam như vậy, làm nhiễu topic quá

P/s: đã gộp, anh cũng đang spam đó ạ, lần này coi như bỏ qua :))


Bài viết đã được chỉnh sửa nội dung bởi MoMo123: 19-04-2018 - 11:50

  N.D.P 

#96
melodias2002

melodias2002

    Trung sĩ

  • Thành viên
  • 105 Bài viết

Bài 40: Cho các số $a,b,c>0$ thoả mãn $a+b+c=3$. Chứng minh rằng $\sum\frac{1}{a^2+2b^2+3}\leq\frac{1}{2}$


Bài viết đã được chỉnh sửa nội dung bởi MoMo123: 19-04-2018 - 11:50


#97
ngominh7s5

ngominh7s5

    Binh nhì

  • Thành viên
  • 12 Bài viết
Mình góp vui 1 bài :).

Bài 41: Cho các số thực dương $a,b,c$ thỏa mãn $a+b+c=1$. Chứng minh rằng
$$ab^2+bc^2+ca^2\le\frac 19.$$

#98
tr2512

tr2512

    Thượng sĩ

  • Thành viên
  • 272 Bài viết

Bài 33:

Cho các số thực không âm a, b, c trong đó không có hai số nào đồng thời bằng 0 và $a+b+c=3$. Chứng minh:

$\frac{a}{{{b^2} + \sqrt {ab} }} + \frac{b}{{{c^2} + \sqrt {bc} }} + \frac{c}{{{a^2} + \sqrt {ac} }} \ge \frac{3}{2}$ (Nguyễn Trung Hiếu)

Lời giải:

Với ý tưởng là AM-GM ngược dấu, ta có:

$\frac{a}{{{b^2} + \sqrt {ab} }} = \frac{{a\sqrt b }}{{{b^2}\sqrt b + b\sqrt a }} = \frac{{\frac{{\sqrt a }}{{\sqrt b }}\left( {b\sqrt a + {b^2}\sqrt b } \right) - {b^2}\sqrt a }}{{{b^2}\sqrt b + b\sqrt a }} = \sqrt {\frac{a}{b}} - \frac{{{b^2}\sqrt a }}{{{b^2}\sqrt b + b\sqrt a }} \ge \sqrt {\frac{a}{b}} - \frac{{{b^2}\sqrt a }}{{2b\sqrt {b\sqrt {ab} } }}\\ = \sqrt {\frac{a}{b}} - \frac{{4\sqrt[4]{{ab}}}}{8} \ge \sqrt {\frac{a}{b}} - \frac{{a + b + 1 + 1}}{8} = \sqrt {\frac{a}{b}} - \frac{{a + b + 2}}{8}$

Lập 2 bất đẳng thức tương tự và sử dụng đánh giá: $\sqrt {\frac{a}{b}} + \sqrt {\frac{b}{c}} + \sqrt {\frac{c}{a}} \ge 3$ Ta thu được điều phải chứng minh.



#99
tr2512

tr2512

    Thượng sĩ

  • Thành viên
  • 272 Bài viết

Mình góp vui 1 bài :).

Bài 41: Cho các số thực dương $a,b,c$ thỏa mãn $a+b+c=1$. Chứng minh rằng
$$ab^2+bc^2+ca^2\le\frac 19.$$

Đề bài không chính xác, bộ số (0.1;0.3;0.6) là một phản ví dụ.



#100
ngominh7s5

ngominh7s5

    Binh nhì

  • Thành viên
  • 12 Bài viết

@tr2512. Đấy là bddt mình thử dùng để giải một bài toán khác nhưng có vẻ như mình chứng minh sai rồi  :luoi: .

 

Bài toán gốc như sau.

 

Cho $a,b,c$ là các số dương thỏa mãn $a+b+c=1$. Tìm giá trị nhỏ nhất của 

$$P=\frac{a}{1+9b^2}+\frac{b}{1+9c^2}+\frac{c}{1+9a^2}.$$







Được gắn nhãn với một hoặc nhiều trong số những từ khóa sau: bất đẳng thức, holder, cosi, bunhiacopxki

2 người đang xem chủ đề

0 thành viên, 2 khách, 0 thành viên ẩn danh